help please! how do I calculate the coordinates of point a so that at is an altitude?

Help Please! How Do I Calculate The Coordinates Of Point A So That At Is An Altitude?

Answers

Answer 1

Part (a)

Since we want segment AT to be an altitude, this means that point A is somewhere on segment RI such that angle TAI is 90 degrees. In other words, segments AT and RI are perpendicular. Check out the diagram below.

We'll need to find the slope of line RI, so we'll need the slope formula

R = (x1,y1) = (-2,-2)

I = (x2,y2) = (4,1)

m = slope of line RI

m = (y2-y1)/(x2-x1)

m = (1-(-2))/(4-(-2))

m = (1+2)/(4+2)

m = 3/6

m = 1/2

m = 0.5

The slope of line RI is 1/2 or 0.5

Now let's use point R(-2,-2) along with that slope we just found to find the equation of line RI.

Turn to point slope form

y - y1 = m(x - x1)

y - (-2) = 0.5(x -(-2))

y + 2 = 0.5(x + 2)

y + 2 = 0.5x + 1

y = 0.5x + 1 - 2

y = 0.5x - 1

This is the equation of line RI. We'll use it later.

----------

We found that the slope of line RI was 1/2.

The negative reciprocal involves us flipping the fraction and flipping the sign to get -2/1 or simply -2.

The perpendicular slope is -2. This is the slope of altitude AT.

We want line AT to go through T(-2,4), so we'll use this point along with the perpendicular slope we just found to calculate the equation of line AT.

We'll turn to the point slope form

y - y1 = m(x - x1)

y - 4 = -2(x - (-2))

y - 4 = -2(x + 2)

y - 4 = -2x - 4

y = -2x - 4 + 4

y = -2x

The equation of the line that goes through points A and T is y = -2x.

----------

To summarize so far, we have these two equations

y = 0.5x - 1 .... equation of line RIy = -2x .... equation of line AT

Apply substitution to solve for (x,y) to find the intersection point. This will determine where point A is located.

y = 0.5x - 1

-2x = 0.5x - 1

-2x-0.5x = -1

-2.5x = -1

x = -1/(-2.5)

x = 0.4

Then we can determine y like so

y = -2x

y = -2(0.4)

y = -0.8

Therefore, point A is located at (0.4, -0.8). Both coordinate values are exact. In fraction form, we can say that the point is (2/5, -4/5), but I find decimals easier to work with in this instance.

Answer:  (0.4, -0.8)

===============================================================

Part (b)

To find the length of segment AT, we can find the distance from A to T. As you can probably guess, we'll use the distance formula.

A = (x1,y1) = (0.4, -0.8) ... found back in part (a)

T = (x2,y2) = (-2, 4) ... given

[tex]d = \sqrt{ (x_2-x_1)^2+(y_2-y_1)^2 }\\\\d = \sqrt{ (-2-0.4)^2+(4 - (-0.8))^2 }\\\\d = \sqrt{ (-2-0.4)^2+(4 + 0.8)^2 }\\\\d = \sqrt{ (-2.4)^2+(4.8)^2 }\\\\d = \sqrt{ 5.76+23.04 }\\\\d = \sqrt{ 28.8 }\\\\d \approx 5.36656\\\\d \approx 5.37\\\\[/tex]

The distance from point A to point T is approximately 5.37 units. This means that segment AT is roughly 5.37 units long.

Answer: 5.37 units
Help Please! How Do I Calculate The Coordinates Of Point A So That At Is An Altitude?

Related Questions

Answer always, sometimes, or never

Answers

Answer:

9. Always

10. Never

11. Always

12. Sometimes

Step-by-step explanation:

Natural numbers are any positive whole numbers (e.g. 1, 2, 3, 4….)Integers are any whole number, positive or negative (e.g. -2, -1, 0, 1, 2…)Rational numbers are any number that can be put into a fraction (or if it’s a decimal, a number that ends)Irrational numbers are any number that cannot be put into a fraction (like a never ending decimal; e.g. pi)Real numbers are any and all numbers (rational or irrationa)

what does this equal 2^3 + 6^5=

Answers

[tex]\\ \sf\longmapsto 2^3+6^5[/tex]

[tex]\\ \sf\longmapsto 2^3+(2\times 3)^5[/tex]

[tex]\\ \sf\longmapsto 8+2^5\times 3^5[/tex]

[tex]\\ \sf\longmapsto 8+32\times 243[/tex]

[tex]\\ \sf\longmapsto 40+7776[/tex]

[tex]\\ \sf\longmapsto 7784[/tex]

Answer:

2*2*2= 8

6*6*6*6*6= 7,776

7,776+8=

7,784

Mr. Clifton was preparing for his upcoming birthday party. He ordered 51 white balloons and 47 red balloons. As he walked to his car 12 balloons flew away! With the remaining balloons, he mixed the colors to create bunches of 3 to decorate his party space. How many full bunches did Mr. Clifton have to decorate his party space?

Answers

Answer:

28 i think

Step-by-step explanation:

51+47=98

98-12 the ones that flew away that takes it to 86

86/3 the 3 balloons buntch than is 28

p.s. i mght of got some equations wrong but this is what i got

Please help! Question is given below in form of image!

Answers

Answer:

c

Step-by-step explanation:

Answer:

None of the choices are correct.

Step-by-step explanation:

[tex] 9x^3 + 9x^2 - 4x - 4 = 0 [/tex]

The polynomial has 4 terms and no common factors. We can try factoring by grouping.

[tex] 9x^2(x + 1) - 4(x + 1) = 0 [/tex]

[tex] (x + 1)(9x^2 - 4) = 0 [/tex]

Now we factor the difference of squares.

[tex] (x + 1)(3x + 2)(3x - 2) = 0 [/tex]

[tex] x + 1 = 0~~or~~3x + 2 = 0~~or~~3x - 2 = 0 [/tex]

[tex]x = -1~~or~~x = -\dfrac{2}{3}~~or~~x = \dfrac{2}{3}[/tex]

None of the choices are correct.

(b) In a sale the original prices are reduced by 15%.
(i)
Calculate the sale price of a book that has an original price of $12

Answers

Answer: $10.20

Step-by-step explanation:

convert the percentage to a decimal: 0.15

multiply the decimal 0.15 by the original price, $12, to find the amount that it will be discounted by: $1.80

subtract the discount from the original price: $10.20

Answer:

$9

Step-by-step explanation:

reduced price=15%*$12

= 15/100*$12

= $3

Sale price=original price - reduced Price

=$12-$3

=$9

Joe is responsible for reserving hotel rooms for a company trip. His company changes plans and increases how many people are going on the trip, so they need at least 50 total rooms. Joe had already reserved and paid for 16 rooms, so he needs to reserve additional rooms. He can only reserve rooms in blocks, and each block contains 8 rooms and costs $900. Let B represent the number of additional blocks that Joe reserves. 1) Which inequality describes this scenario? 2)What is the least amount of money that Joe can spend to get the amount of rooms they need? Work not needed!

Answers

Answer:

The answer is below

Step-by-step explanation:

1) The number of rooms Joe company needs is 50. Let the number of rooms needed be n, this can be represented by the inequality:

n ≥ 50

Joe has already reserved 16 rooms, therefore the number of additional rooms needed to be reserved = 50 - 16 = 34 rooms. At least 34 rooms have to be reserved, if B is the number of additional blocks that Joe reserves, the inequality is:

B ≥ 34

2) Each block contains 8 rooms therefore the minimum number of block (a) needed to be reserved = 34/8 = 4.25 = 5 to the next whole number. Therefore the minimum number of blocks needed is given as:

a ≥ 5

Since each block cost $900, let c represent the minimum amount of money needed, therefore the least amount of money needed is given as:

c = $900(5) = $4500

c ≥ $4500

Here, we are required to determine which inequality describes the scenario in the question and the least amount of money that Joe can spend to get the amount of rooms they need.

(1) The inequality which best describes the scenario is; B ≥ 4.25.

(2) The least amount of money that Joe can spend to get the amount of rooms they need is; $4500

First, since they need at least 50 total rooms and Joe had already reserved 16 rooms.

Therefore, Joe needs to reserve an additional 34 rooms to completely accomodate people who are going on the trip.

And since, he can only reserve rooms in blocks with each block containing 8 rooms;

The number of blocks he needs to reserve; B ≥ 4.25.

Therefore, since only whole blocks can be reserved, Joe needs to reserve 5 blocks.

Therefore, since one block costs $900, then, 5 blocks will cost $900 × 5 = $4500.

Read more:

https://brainly.com/question/19003099

What effect will replacing x with (x – 7)have on the graph of the equation y = = (x + 4)??
A. slides the graph 3 units down
B. slides the graph 3 units up
C. slides the graph 7 units right
D. slides the graph 3 units right

Answers

Answer:

D

Step-by-step explanation:

When you graph the first equation y = (x + 4) it will have a x-intercept of -4 and a y-intercept of 4.  

When you replace x with (x-7) the equation will be converted into

y = ((x-7)+4)

y = x-3

When you graph this you see it has a x-intercept of 3 and a y-intercept of -3

To find the distance add the 2 x values together:

-4 + (-3) = ?

-4 -3 = ?

-7 = ?

Therefore the graph will shift units right

The weight of a small starbucks coffee is a normally distributed random variable with a mean of 325 grams and a standard deviation of 10 grams. Find the weight that corresponds to each event (round your final answers to 2 decimal places)

Answers

The weight of a small starbucks coffee is a normally distributed random variable with a mean of 325 grams and a standard deviation of 10 grams. Find the weight that corresponds to each event (round your final answers to 2 decimal places)

a. Highest 10 percent

b. Middle 50 percent

Answer:

the weight that corresponds to  Highest 10% = 337.8

the weight that corresponds to  Middle 50 % lies between 318.26  and 331.74

Step-by-step explanation:

From the information provided for us:

we have the mean = 325

the standard deviation = 10

The objective is to find the weight that corresponds to each event  i.e for event (a) , highest 10%

So;

The probability of P (Z > z) = 10%

Same as:

0.1 = 1 - P( Z < z)

P( Z < z) = 1 - 0.1

P( Z < z) = 0.9

From the standard normal tables for z;

P( Z < 1.28) = 0.9

z = 1.28

Similarly. from the z formula; we have:

[tex]z = \dfrac{X - \mu}{\sigma}[/tex]

[tex]z \times \sigma = X - \mu[/tex]

[tex]z \times \sigma + \mu= X[/tex]

[tex]X= z \times \sigma + \mu[/tex]

X = (1.28 × 10) + 325

X = 12.8 + 325

X = 337.8

Therefore, the weight that corresponds to  Highest 10% = 337.8

b.  the weight that corresponds to  Middle 50 % can be computed as follows:

the region of z values at 0.50 lies between -0.674 and +0.674

from the z formula; we have:

[tex]z = \dfrac{X - \mu}{\sigma}[/tex]

[tex]z \times \sigma = X - \mu[/tex]

[tex]z \times \sigma + \mu= X[/tex]

[tex]X= z \times \sigma + \mu[/tex]

X = -0.674 × 10 + 325  and  X = 0.674 × 10 + 325

X = - 6.74 + 325   and      X = 6.74  + 325

X = 318.26  and    X = 331.74

the weight that corresponds to  Middle 50 % lies between 318.26  and 331.74

A candy store sells only one type of candy, which comes in different colors. Each color is a different flavor, and some flavors cost more than others. Specifically, 1 red piece costs the same as 3 blue pieces, 2 orange pieces cost the same as 5 green pieces, and 2 blue pieces cost the same as 10 green pieces. If 1 orange piece costs 13 cents, how much will 8 red pieces cost?

Answers

Answer:

$6.24

Step-by-step explanation:

1 orange=%0.13

2 orange=5 green=$0.26

10 green=2 blue= $0.52

.52+.26=.78

1 red=3 blue=$0.78

0.78*8=6.24

You pay $10 to play the following game of chance. there is a bag containing 12 balls: three are red, five are green, and the rest are yellow. You are to draw one ball from the bag. You will win $15 if you draw a red ball, and you will win $10 if you draw a yellow ball. You win nothing for drawing a green ball. What is the expected value? Should you play?​

Answers

Answer:

$ -2.08  expected to lose  

Step-by-step explanation:

3 25.0%  $15.00   $5.00   $1.25  

5 41.7%  $10.00   $-     $-    

4 33.3%  $-     $(10.00)  $(3.33)  

     

    $(2.08)  expected to lose  

How do I find the Derivative of a Function where the x is a number?

If I was to find d/dx g(1) for example, would I first find the value of the function g(x) at x=1 and then find the derivative of that which becomes 0 because the result would probably be a constant. Or do I first find the derivative of g(x) and then plug in x=1 to calculate it?

Thanks

Answers

Given a function g(x), its derivative, if it exists, is equal to the limit

[tex]g'(x) = \displaystyle\lim_{h\to0}\frac{g(x+h)-g(x)}h[/tex]

The limit is some expression that is itself a function of x. Then the derivative of g(x) at x = 1 is obtained by just plugging x = 1. In other words, find g'(x) - and this can be done with or without taking a limit - then evaluate g' (1).

Alternatively, you can directly find the derivative at a point by computing the limit

[tex]g'(1) = \displaystyle\lim_{h\to0}\frac{g(1+h)-g(1)}h[/tex]

But this is essentially the same as the first method, we're just replacing x with 1.

Yet another way is to compute the limit

[tex]g'(1) = \displaystyle\lim_{x\to1}\frac{g(x)-g(1)}{x-1}[/tex]

but this is really the same limit with h = x - 1.

You do not compute g (1) first, because as you say, that's just a constant, so its derivative is zero. But you're not concerned with the derivative of some number, you care about the derivative of a function that depends on a variable.

Solve logs (8 - 3x) = log20 for x.
A. X = 14
B. X = -13
C.x = -8
D. X= -4

Answers

Answer:

x = -4

Step-by-step explanation:

logs (8 - 3x) = log20

Since we are taking the log on each side

log a = log b  then a = b

8 -3x = 20

Subtract 8  from each side

8 -3x-8 =20 -8

-3x = 12

Divide by -3

-3x/-3 = 12/-3

x = -4

Answer:

[tex] \boxed{\sf x = -4} [/tex]

Step-by-step explanation:

[tex] \sf Solve \: for \: x \: over \: the \: r eal \: numbers:[/tex]

[tex] \sf \implies log(8 - 3x) = log 20[/tex]

[tex] \sf Cancel \: logarithms \: by \: taking \: exp \: of \: both \: sides:[/tex]

[tex] \sf \implies 8 - 3x = 20[/tex]

[tex] \sf Subtract \: 8 \: from \: both \: sides:[/tex]

[tex] \sf \implies 8 - 3x - 8 = 20 - 8 [/tex]

[tex] \sf \implies - 3x = 12 [/tex]

[tex] \sf Divide \: both \: sides \: by \: - 3:[/tex]

[tex] \sf \implies \frac{-3x}{-3} = \frac{12}{-3} [/tex]

[tex] \sf \implies x = - 4[/tex]

Solve the following non-linear system of equations algebraically.
y = - 2x² - 4x + 5 and
y = 2x^2 + 4x + 5

Answers

Answer:

x = -2

Step-by-step explanation:

since y is common in both equations, you can equate them to be equal

-2x² - 4x + 5 = 2x² + 4x + 5

4x² + 8x = 0

factor out 4x

4x(x + 2) = 0

divide both sides by 4x

x + 2 = 0

x = -2

At dinner, 100 students pass through the cafeteria line and were served meals. 40 fish entrees and 60 pasta entrees were served to the students. A total of 20 students chose neither entree. Assuming all students were served zero, one, or two entrees, how many students were served two entrees

Answers

Answer:  20

Step-by-step explanation:

Given: Total students at the dinner = 100

Number of fish entrees = 40

Number of pasta entrees = 60

Number of students chose neither entree = 20

Now , Number of students chose either fish or pasta = (Total students) - (Number of students chose neither entree)

= 100-20

= 80

Now , Number of students chose either fish or pasta = (Number of fish entrees) + (Number of pasta entrees)- (Number of students chose both)

⇒ Number of students chose both = (Number of fish entrees) +(Number of pasta entrees)-(Number of students chose either fish or pasta)

= 40+60-80

= 20

Hence, the number of students were served two entrees = 20

20 students were served two entrees.

Given,

total student pass through cafeteria line and were served meal is 100.

No. of students choose fish entries is 40.

No. of students choose pasta entrees is 60.

No. of student choose neither entree is 20.

We have to calculate the no. of students served two entrees.

Now Number of students chose either fish or pasta will be,

[tex]N=100-20[/tex]

[tex]N=80[/tex]

Now no. of students choose both will be,

[tex]N=(fish\ entree+\ pasta \ entree )-Entree\ either \ pasta \ or \ fish[/tex]

[tex]N=60+40-80[/tex]

[tex]N=20[/tex]

Hence 20 students were served two entrees.

For more details follow the link:

https://brainly.com/question/24550771

Need help ! Help needed very much !

Answers

Answer:

x/2 > 5

Step-by-step explanation:

Quotient means division

x/2

is more than means greater

x/2 > 5

how do you solve this?

Answers

Triangles ABC and LBM are similar. We know this because AL and LB have the same length, so that AB is twice as long as either AL or LB. The same goes for MC and BM, and BC. The angle B is the same for both tirangles ABC and LBM, so the side-angle-side postulate tells us the triangles are similar, and in particular that triangle ABC is twice as large as LBM.

All this to say that LM must be half as long as AC, so LM has length (B) 14 cm.

The sum of three prime numbers is 22. What is the largest possible product of these numbers?

Answers

Step-by-step explanation:

a+b+c=22

abc=?

2+3+17=22

2×3×17=102

Answer:

The numbers are 2, 7 and 13

Step-by-step explanation:

2+7+13 = 22

2X7X13 = 182

VW = 40 in. The radius of the circle is 25 inches. Find
the length of SP.
С.
P
T
V
W

Answers

Answer:

I think The choose C. 15

Step-by-step explanation:

wv=40

vG=20 =ST

PT=25

ST²+SP²=PT² —> 20²+SP²=25 —> 400+SP²=625

PS²=225 —> SP=15 in

I hope I helped you^_^

The value of SP is 15 inch.

What is Pythagoras theorem?

The well-known geometric theorem that the sum of the squares on the legs of a right triangle is equal to the square on the hypotenuse.

Given:

VW=40

VG=20 =ST

and, PT=25

Now, using pythagoras theorem

ST²+SP²=PT²

20²+SP²=25

400+SP²=625

PS²=225

SP=15 in

Learn more about pythagoras theorem here:

https://brainly.com/question/343682

#SPJ5

Helppp meeeew pleaseeeee

Answers

Answer:

Hey there!

1 1/10=1.1

2/25=0.08

Each serving requires 0.08 kg, and he has 1.1 kg.

Thus, he can make 1.1/0.08, or 13.75 servings.

Let me know if this helps :)

Answer:

13 servings of tofu dish.

Step-by-step explanation:

First, convert the fractions to decimal numbers:

1 1/10 kg = 0.1 kg

2/25 kg = 0.08 kg

Now find how many servings of tofu dish will cover 0.1 kg of tofu:

2/25 kg = 1 serving

1 1/10 kg = ?

= 1 1/10 ÷ 2/25 kg

= 11/10 × 25/2

= 55/4

= 13.75 servings

Approximate it to a whole number:

13 servings.

Use the motion map to answer the question.
Which scenario could be represented by the motion
map?

O A car speeds up to merge onto the freeway and
then continues at a constant velocity
O A car speeds up to pass a truck, then slows down
to a constant velocity.
O A car slows to stop at a stop sign. Once traffic is
clear, the car speeds up.
O A car slows to makes a U-turn, then continues in
the opposite direction.

Answers

Answer:

A car slows to stop at a stop sign. Once traffic is clear, the car speeds up.

Step-by-step explanation:

Answer:

C.) A car slows to stop at a stop sign. Once traffic is clear, the car speeds up.

Step-by-step explanation:

. Find two polynomial expressions whose quotient, when simplified, is 1/x . Use that division problem to determine whether polynomials are closed under division.

Answers

Answer:

The two polynomials are:

(x + 1) and (x² + x)

Step-by-step explanation:

A polynomial is simply an expression which consists of variables & coefficients involving only the operations of addition, subtraction, multiplication, and non - negative integer exponents of variables.

Now, 1 and x are both polynomials. Thus; 1/x is already a quotient of a polynomial.

Now, to get two polynomial expressions whose quotient, when simplified, is 1/x, we will just multiply the numerator and denominator by the same polynomial to get more quotients.

So,

Let's multiply both numerator and denominator by (x + 1) to get;

(x + 1)/(x(x + 1))

This gives; (x + 1)/(x² + x)

Now, 1 and x are both polynomials but the expression "1/x" is not a polynomial but a quotient and thus polynomials are not closed under division.

PLS HELP! Using the diagram shown below, which statement is true?

Answers

Your answer is : the measure of

Tan x is ........ Cos x is ........ Sin x is ..........

Answers

Answer:

Tan x= 3/4 Cos x= 4/5 Sin x=3/5

Step-by-step explanation:

"soh-cah-toa"

Just an easy way to remember how Sine, Cosine and Tangent work:

Soh...

Sine = Opposite / Hypotenuse

...cah...

Cosine = Adjacent / Hypotenuse

...toa

Tangent = Opposite / Adjacent

Answer:

tan(x)=0.75 sin(x)=0.6 Cos(x)=0.8

Step-by-step explanation:

[tex]tan(x)=\frac{opposite}{adjacent}=\frac{3}{4}=0.75[/tex]

[tex]cos(x)=\frac{adjacent}{hypotenuse}= \frac{4}{5} =0.8[/tex]

[tex]sin(x)=\frac{opposite}{hypotenuse}=\frac{3}{5}=0.6[/tex]

opposite=3

adjacent=4

hypotenuse=5

try remembering it as SOH CAH TOA

Rejoice bought 600 oranges at 5 for GH¢3.00 to be sold at the market. On her arrival 5% of the oranges got rotten and she sold the rest at one for GH¢1.00...
I) How any oranges did she finally sell?
ii) Find her loss or profit percent.​

Answers

Answer:

She finally sold 570 oranges

Profit %= 58.33%

Step-by-step explanation:

Quantity bought=600

Price=5 for GH¢3.00

Total cost price=600/5 * GH¢3.00

=120*GH¢3.00

=GH¢360.00

5% of 600 oranges got rotten

=5/100*600

=30 Oranges were rotten

I) How any oranges did she finally sell?

She finally sold

Sold oranges= Total oranges - Rotten oranges

=600-30

=570 oranges

Selling price=GH¢1.00 * 570 oranges

=GH¢570.00

ii) Find her loss or profit percent

Profit or loss percent= Selling price - cost price / cost price * 100

% profit or loss=S.P - C.P / C.P * 100

=GH¢570.00 - GH¢360.00 / GH¢360.00 * 100

=GH¢210.00/GH¢360.00 *100

=0.5833 * 100

=58.33% profit

Plz help THank you Image is proved Below

Answers

Answer:

22 is the correct answer of your question hope it helps you

11) MZHGF = 16x + 4, m EGF = 110°,
and MZHGE = 3x + 11. Find x.
G
H
F
E

Answers

Answer:  x = 9

=======================================================

Explanation:

The smaller angles HGE and EGF combine to form the largest angle HGF.

This is an example of the angle addition postulate.

(angle HGE) + (angle EGF) = angle HGF

(3x+11) + (110) = 16x+4

3x+121 = 16x+4

121-4 = 16x-3x

117 = 13x

13x = 117

x = 117/13

x = 9 which is the final answer

We can stop here.

--------------------

Extra info (optional section)

Use this x value to find each angle shown below

angle HGE = 3x+11 = 3*9+11 = 27+11 = 38 degreesangle HGF = 16x+4 = 16*9+4 = 144+4 = 148 degrees

Then notice how,

(angle HGE)+(angle EGF) = 38+110 = 148

which is exactly the measure of angle HGF

This confirms that the equation

(angle HGE) + (angle EGF) = angle HGF

is true for that x value of x = 9. Therefore, the answer is confirmed.

9514 1404 393

Answer:

  11) x = 9°

  13) x = -12°

Step-by-step explanation:

11) ∠HGE +∠EGF = ∠HGF

  (3x +11) +(110°) = (16x +4)

  117° = 13x . . . . . . . . . . . . . . . subtract 3x+4, collect terms

  9° = x . . . . . . . . . . . . divide by 13

__

13) ∠BCF +∠FCD = ∠BCD

  (x +78) +(x +41) = 95°

  2x = -24°

  x = -12°

AB = 3.2 cm
BC= 8.4 cm
The area of triangle ABC is 10 cm²
Calculate the perimeter of triangle ABC.
Give your answer correct to three significant figures.​

Answers

Answer:

Therefore, perimeter of the given triangle is 18.300 cm.

Step-by-step explanation:

Area of the triangle ABC = [tex]\frac{1}{2}(\text{AB})(\text{BC})(\text{SinB})[/tex]

10 = [tex]\frac{1}{2}(3.2)(8.4)(\text{SinB})[/tex]

Sin(B) = [tex]\frac{10}{3.2\times 4.2}[/tex]

B = [tex]\text{Sin}^{-1}(0.74405)[/tex]

B = 48.08°

By applying Cosine rule in the given triangle,

(AC)² = (AB)² + (BC)²-2(AB)(BC)CosB

(AC)² = (3.2)² + (8.4)² - 2(3.2)(8.4)Cos(48.08)°

(AC)² = 10.24 + 70.56 - 35.9166

(AC)² = 44.88

AC = [tex]\sqrt{44.8833}[/tex]

AC = 6.6995 cm

Perimeter of the ΔABC = m(AB) + m(BC) + m(AC)

                                      = 3.200 + 8.400 + 6.6995

                                      = 18.2995

                                      ≈ 18.300 cm

Therefore, perimeter of the given triangle is 18.300 cm

11,9,7,5,3,1,
B) Common Difference:
Recursive Function:
D) Explicit
Function:​

Answers

Answer:

The terms 11, 9, 7, 5, 3, 1 have a common difference of -2 therefore, the correct option defining the relationship between the terms is

B) Common difference

Step-by-step explanation:

The common difference between a series of numbers is found by subtracting a number from the next number following and a common difference exists  when the difference between successive adjacent number pairs is the same

A sequence that has a common difference is an arithmetic sequence or arithmetic projection.

The given sequence, 11, 9, 7, 5, 3, 1, is an arithmetic sequence.

How to solve this question pls help asap

Answers

Answer:

answered by g a u t h m a t h

Step-by-step explanation:

(-9) - 9n= (-45) please helpppp!

Answers

Answer: n=4

Step-by-step explanation:

[tex]\left(-9\right)-9n=\left(-45\right)[/tex]

add 9 to both sides

[tex]-9-9n+9=-45+9[/tex]

[tex]-9n=-36[/tex]

divide both sides by -9

[tex]n=4[/tex]

Brainliest please

━━━━━━━☆☆━━━━━━━

▹ Answer

n = 4

▹ Step-by-Step Explanation

(-9) - 9n = (-45)

Simple terms are written last:

-9 - 9 = -45

Group all the variable terms on one side, and all the constant terms on the other side:

(-9n - 9) + 9 = -45 + 9

n = 4

Hope this helps!

CloutAnswers ❁

━━━━━━━☆☆━━━━━━━

Other Questions
13 Water is pulled up from a well in a bucket on a rope. The rope winds on a cylindrical drum 15 cm in diameter. It takes 28 turns of the drum to pull the bucket up from the bottom of the well. How deep is the well? (Use the value 2 for m.) What did the Sherman anti-trust act of 1890 intend to limit (All yes or no questions). Determine whether each of the following pairs of angles have equal measures:a. KJLand LJMb. MJP and PJRC. LJP and NJRd. MJK and PJR An entry in the Peach Festival Poster Contest must be rectangular and have an area of 1200 square inches. Furthermore, it's length must be 20 inches longer than it's width. Find the dimensions. Three friends are at a carnival. Beth goes on 2 rides for $5.00. Zane goes on 5 rides for $12How much does it cost Tina to go on 9 rides?.50. Martin currently has a balance of $948 in an account he has held for 20 years. He opened the account with an initial deposit of $600. What is the simple interest on the account?A - 1.8% B - 2.9%C - 3.2% D - 7.9% Chelsi has talked to her artist friends about how much money they earn each year from working in the arts. She gathers these values from seven people: [$1,500; $6,700; $2,200; $8,100; $50,500; $12,000; $2,200].What is the median of this data set? Help! A sales person lying about the effectiveness of his product would be an example of witch of the following?A: direct persuasion B: professional persuasion C: emotional persuasion D: indirect persuasion E: unethical persuasion Ethic of care refers to a. the ethics associated with choosing between ones leisure and ones family b. obligations to care for ones family c. the ethics associated with "women belong in the kitchen" d. obligations to ensure we take care of ourselves Sheffield Corporation purchased machinery on January 1, 2017, at a cost of $250,000. The estimated useful life of the machinery is 4 years, with an estimated salvage value at the end of that period of $24,000. The company is considering different depreciation methods that could be used for financial reporting purposes.Required:Prepare separate depreciation schedules for the machinery using the straight-line method, and the declining-balance method using double the straight-line rate. As director of a positive youth development program working with teens who have been expelled from school for violent behavior, what would be the main focus of your staff's activities I need help with this ASAP!Find the area of the sector. The Whistling Straits Corporation needs to raise $74 million to finance its expansion into new markets. The company will sell new shares of equity via a general cash offering to raise the needed funds. The offer price is $45 per share and the company's underwriters charge a spread of 6 percent. If the SEC filing fee and associated administrative expenses of the offering are $825,000, how many shares need to be sold? (Do not round intermediate calculations and enter your answer in dollars, not millions, rounded to the nearest whole number, e.g., 1,234,567.) The perpendicular bisector of the line segment connecting the points $(-3,8)$ and $(-5,4)$ has an equation of the form $y = mx + b$. Find $m+b$. what are the features of humanism? 5. solve for x please help A sample of 81 observations is taken from a normal population with a standard deviation of 5. The sample mean is 40. Determine the 95% confidence interval for the population mean. what is called the priest of magar? Given two points M & N on the coordinate plane, find the slope of MN , and state the slope of the line perpendicular to MN . (there's two questions) 1) M(9,6), N(1,4) 2) M(-2,2), N(4,-4) PLEASE HELP!!! 10. Write the formula of the function f(x) whose graph is shown.Af(x) = 4 - 4Bf(x)=1= - +4Xf(x) = 24D1f(x) =X +4